Menu Close

e-2-2-3-3-4-4-5-5-




Question Number 140506 by Dwaipayan Shikari last updated on 08/May/21
e^((((ζ(2))/2)−((ζ(3))/3)+((ζ(4))/4)−((ζ(5))/5)+...))    =?
$${e}^{\left(\frac{\zeta\left(\mathrm{2}\right)}{\mathrm{2}}−\frac{\zeta\left(\mathrm{3}\right)}{\mathrm{3}}+\frac{\zeta\left(\mathrm{4}\right)}{\mathrm{4}}−\frac{\zeta\left(\mathrm{5}\right)}{\mathrm{5}}+…\right)} \:\:\:=? \\ $$
Answered by mnjuly1970 last updated on 08/May/21
Commented by mnjuly1970 last updated on 08/May/21
    Hi  mr payan..       please check my solution....
$$\:\:\:\:{Hi}\:\:{mr}\:{payan}.. \\ $$$$\:\:\:\:\:{please}\:{check}\:{my}\:{solution}…. \\ $$$$\:\:\:\:\: \\ $$
Commented by Dwaipayan Shikari last updated on 08/May/21
Correct sir ! Great!
$${Correct}\:{sir}\:!\:{Great}! \\ $$
Commented by Niiicooooo last updated on 08/May/21
nice proof!
$${nice}\:{proof}! \\ $$
Commented by mnjuly1970 last updated on 08/May/21
  mercey...
$$\:\:{mercey}… \\ $$
Answered by Dwaipayan Shikari last updated on 08/May/21
ψ(z+1)=ψ(1)+((ψ′(1))/(1!))z+((ψ′′(1))/(2!))z^2 +...  ⇒ψ(z+1)=−γ+ζ(2)z−ζ(3)z^2 +ζ(4)z^3 −...  ⇒∫_0 ^1 ψ(z+1)dz=−γ+∫_0 ^1 ζ(2)z−ζ(3)z^2 +..dz  ⇒[log(Γ(z+1))]_0 ^1 +γ=((ζ(2))/2)−((ζ(3))/3)+((ζ(4))/4)−..  γ=((ζ(2))/2)−((ζ(3))/3)+...
$$\psi\left({z}+\mathrm{1}\right)=\psi\left(\mathrm{1}\right)+\frac{\psi'\left(\mathrm{1}\right)}{\mathrm{1}!}{z}+\frac{\psi''\left(\mathrm{1}\right)}{\mathrm{2}!}{z}^{\mathrm{2}} +… \\ $$$$\Rightarrow\psi\left({z}+\mathrm{1}\right)=−\gamma+\zeta\left(\mathrm{2}\right){z}−\zeta\left(\mathrm{3}\right){z}^{\mathrm{2}} +\zeta\left(\mathrm{4}\right){z}^{\mathrm{3}} −… \\ $$$$\Rightarrow\int_{\mathrm{0}} ^{\mathrm{1}} \psi\left({z}+\mathrm{1}\right){dz}=−\gamma+\int_{\mathrm{0}} ^{\mathrm{1}} \zeta\left(\mathrm{2}\right){z}−\zeta\left(\mathrm{3}\right){z}^{\mathrm{2}} +..{dz} \\ $$$$\Rightarrow\left[{log}\left(\Gamma\left({z}+\mathrm{1}\right)\right)\right]_{\mathrm{0}} ^{\mathrm{1}} +\gamma=\frac{\zeta\left(\mathrm{2}\right)}{\mathrm{2}}−\frac{\zeta\left(\mathrm{3}\right)}{\mathrm{3}}+\frac{\zeta\left(\mathrm{4}\right)}{\mathrm{4}}−.. \\ $$$$\gamma=\frac{\zeta\left(\mathrm{2}\right)}{\mathrm{2}}−\frac{\zeta\left(\mathrm{3}\right)}{\mathrm{3}}+… \\ $$
Commented by mnjuly1970 last updated on 08/May/21
nice very nice...
$${nice}\:{very}\:{nice}… \\ $$

Leave a Reply

Your email address will not be published. Required fields are marked *